What is the correlation of X and Z in terms of variance and covariance?

Click For Summary
The discussion focuses on calculating the covariance and correlation coefficient between independent random variables X and Z, where Z is defined as Z = X - Y. The variances of X and Y are given as 9 and 7, respectively, leading to a variance of Z calculated as 16. The covariance Cov(X,Z) is derived using the properties of covariance, noting that Cov(X,Y) equals zero due to their independence. The correlation coefficient is expressed in terms of Cov(X,Z) and the square roots of the variances of X and Z. The conversation emphasizes understanding the covariance definition to clarify the calculations.
CescGoal
Messages
3
Reaction score
0
Very sorry that I've double posted but I realized i placed the original post in Precalculus.



1. Homework Statement
Question

Let X and Y be independent random variables with variances 9 and 7 respectively and let
Z = X - Y

a) What is the value of Cov(X,Z)
b) What is the value of the correlation coefficient of X and Z?

I've been stuck on this one question for 2-3 hours; its ridiculous, I know. Here's my terrible try.


3. The Attempt at a Solution
a)

Var(X) = 9
Var(Y) = 7

Var(X-Y) = Var(X) + Var(Y) = Var(Z)
Therefore, Var(Z) = 7 + 9 =16
Cov(X,Z) = E[XZ] - E[X]E[Z]


and b) \rhoXZ = \frac{Cov(X,Z)}{\sqrt{Var(X)*Var(Z)}}



= \frac{Cov(X,Z)}{\sqrt{9}*\sqrt{16}}
= \frac{Cov(X,Z)}{12}

Since last topic, I've realized that Cov(X,Y) = 0 due to independency. But I don't know how to use it.
 
Physics news on Phys.org


Cov(X,Z)=Cov(X,X-Y). Cov(X,X-Y)=Cov(X,X)-Cov(X,Y), right?
 


I didn't know about that law; thankyou very much.
 


CescGoal said:
I didn't know about that law; thankyou very much.

It's pretty obvious if you write out the definition of Cov. You should try and do that so you can see why it's true.
 
Question: A clock's minute hand has length 4 and its hour hand has length 3. What is the distance between the tips at the moment when it is increasing most rapidly?(Putnam Exam Question) Answer: Making assumption that both the hands moves at constant angular velocities, the answer is ## \sqrt{7} .## But don't you think this assumption is somewhat doubtful and wrong?

Similar threads

  • · Replies 2 ·
Replies
2
Views
6K
  • · Replies 2 ·
Replies
2
Views
1K
  • · Replies 8 ·
Replies
8
Views
2K
  • · Replies 1 ·
Replies
1
Views
2K
  • · Replies 27 ·
Replies
27
Views
4K
Replies
9
Views
4K
  • · Replies 4 ·
Replies
4
Views
2K
Replies
2
Views
2K
  • · Replies 4 ·
Replies
4
Views
2K
  • · Replies 2 ·
Replies
2
Views
2K